A difficult series expansion (finding a limit)

Click For Summary
SUMMARY

The limit lim_{x \to 0} (ln(1+x^2))/(1-cos(x)) evaluates to 2 using series expansions and L'Hôpital's rule. The Taylor series for ln(1+x^2) and 1-cos(x) are essential for this calculation. By factoring out x^2 from both the numerator and denominator, the limit simplifies to 2. This approach clarifies the indeterminate forms encountered during differentiation.

PREREQUISITES
  • Understanding of Taylor series expansions
  • Familiarity with L'Hôpital's rule
  • Knowledge of limits in calculus
  • Basic differentiation techniques
NEXT STEPS
  • Study Taylor series for common functions like ln(x) and cos(x)
  • Practice applying L'Hôpital's rule to various indeterminate forms
  • Explore series expansion techniques for limits
  • Learn how to manipulate series notation effectively
USEFUL FOR

Students in calculus, mathematicians working with limits and series, and educators teaching series expansions and differentiation techniques.

TheSodesa
Messages
224
Reaction score
7

Homework Statement



Find \lim_{x \to 0}\frac{ln(1+x^2)}{1-cos(x)} by using series representations. Check using L'Hospitals rule.

Homework Equations



Taylor polynomial at x=0: \sum_{k=0}^{\infty}\frac{f^{k}(0)}{k!}(x)^{k} = f(0) + f'(0)(x) + f''(0)x^{2} +...

The Attempt at a Solution



Using L'Hospital's rule gives me a limit of 2. Wolfram Alpha gives the following series expansion for the quotient: http://www.wolframalpha.com/input/?i=series+(ln(1+x^2))/(1-cosx), which obviously evaluates to 2 when x \to 0

What I don't get is how the program calculates the series. Differentiating the quotient given in the problem statement (to find out values of the derivatives to plug into the Taylor-formula) gives very complicated results that either equal zero or are indeterminate when x = 0.

For example, \frac{d}{dx} \frac{ln(1+x^2)}{1-cos(x)} = \frac{2x(1+x^{2})^{-1}}{1-cos(x)} - \frac{ln(1+x^{2})sin(x)}{(1-cos(x))^{2}} where both terms are indeterminate when x = 0. The same thing happens with the second derivative: http://www.wolframalpha.com/input/?i=second+derivative+(ln(1+x^2))/(1-cosx)

So there are indeterminates where things would otherwise seem to evaluate to something that resembles 2, and zeroes everywhere else. It also doesn't help that f(0), the first term in the series, is again indeterminate.

I really have no idea how to approach this problem beyond what I've already tried (which in addition to this approach included plugging in the series for ln(1+x2) and 1-cos(x) in their respective places in the fraction). Could someone point me in the right direction?
 
Last edited:
Physics news on Phys.org
TheSodesa said:
What I don't get is how the program calculates the series. Differentiating the quotient given in the problem statement (to find out values of the derivatives to plug into the Taylor-formula) gives very complicated results that either equal zero or are indeterminate when x = 0.

For example, \frac{d}{dx} \frac{ln(1+x^2)}{1-cos(x)} = \frac{2x(1+x^{2})^{-1}}{1-cos(x)} - \frac{ln(1+x^{2})sin(x)}{(1-cos(x))^{2}} where both terms are indeterminate when x = 0. The same thing happens with the second derivative: http://www.wolframalpha.com/input/?i=second+derivative+(ln(1+x^2))/(1-cosx)
You don't need to evaluate the series of the whole quotient. First write down each of the Taylor expansion of the numerator and denominator separately.
 
blue_leaf77 said:
You don't need to evaluate the series of the whole quotient. First write down each of the Taylor expansion of the numerator and denominator separately.
I tried doing that as well, but a similar issue arises:

If

around x = 1: ln(1+x^{2}) = \sum_{k=1}^{\infty} \frac{(-1)^{k+1}(k-1)!\cdot x^{2k}}{k!}<br /> <br /> = -\sum_{k=1}^{\infty} \frac{(-1)^{k}(k-1)!\cdot x^{2k}}{k!}<br /> <br /> =-\sum_{k=1}^{\infty} \frac{(-1)^{k}\cdot x^{2k}}{k}

and

around x = 0: cos(x) = \sum_{k=0}^{\infty} \frac{(-1)^{k}x^{2k}}{(2k)!}

then

<br /> \lim_{x \to 0} \frac{ln(1+x^{2})}{1-cos(x)}<br /> <br /> = \lim_{x \to 0} \frac{ \sum_{k=1}^{\infty} \frac{(-1)^{k+1}x^{2k}}{k}}{1 - \sum_{k=0}^{\infty} \frac{(-1)^{k}x^{2k}}{(2k)!}}<br /> <br /> = \lim_{x \to 0} \frac{ -\sum_{k=1}^{\infty} \frac{(-1)^{k}x^{2k}}{k}}{1 - \sum_{k=0}^{\infty} \frac{(-1)^{k}x^{2k}}{(2k)!}}<br /> <br /> = \lim_{x \to 0} \frac{ -\sum_{k=1}^{\infty} \frac{(-1)^{k}x^{2k}}{k}}{1 - ( 1 + \sum_{k=1}^{\infty} \frac{(-1)^{k}x^{2k}}{(2k)!})}<br /> <br />
where the numerator and denominator still both evaluate to zero (since I can't just cancel the xs because of the series notation). Therefore I'm still stuck with an indeterminate form.
 
It may be more transparent if I write the expansions explicitly for first few terms
$$
\frac{\ln(1+x^{2})}{1-\cos(x)} = \frac{ x^2-\frac{x^4}{2}+\frac{x^6}{3}-\cdots }{ \frac{x^2}{2!}-\frac{x^4}{4!}+\frac{x^6}{6!}\cdots}
$$
Now what happens if you factor out ##x^2## from the numerator and denominator?
 
blue_leaf77 said:
It may be more transparent if I write the expansions explicitly for first few terms
$$
\frac{\ln(1+x^{2})}{1-\cos(x)} = \frac{ x^2-\frac{x^4}{2}+\frac{x^6}{3}-\cdots }{ \frac{x^2}{2!}-\frac{x^4}{4!}+\frac{x^6}{6!}\cdots}
$$
Now what happens if you factor out ##x^2## from the numerator and denominator?

Well, gee-whiz.

If you factor out x2 and take the limit as x \to 0, we are left with \frac{1}{\frac{1}{2!}} = \frac{2!}{1} = 2.

Thanks a bunch. I'm not used to working with series notation and have been simply thrown in the middle of it all. I should probably start writing the first few terms of a series whenever I'm faced with a problem like this.
 
Question: A clock's minute hand has length 4 and its hour hand has length 3. What is the distance between the tips at the moment when it is increasing most rapidly?(Putnam Exam Question) Answer: Making assumption that both the hands moves at constant angular velocities, the answer is ## \sqrt{7} .## But don't you think this assumption is somewhat doubtful and wrong?

Similar threads

Replies
5
Views
2K
  • · Replies 7 ·
Replies
7
Views
2K
  • · Replies 7 ·
Replies
7
Views
2K
  • · Replies 14 ·
Replies
14
Views
2K
  • · Replies 1 ·
Replies
1
Views
2K
Replies
6
Views
2K
  • · Replies 7 ·
Replies
7
Views
2K
Replies
2
Views
2K
  • · Replies 1 ·
Replies
1
Views
2K
  • · Replies 27 ·
Replies
27
Views
3K